Théorème fondamental des gradients et équation de Lenz-Faraday

Tibsoo
Modifié (November 2022) dans Mathématiques et Physique
Bonjour, 

Dans le livre de Griffiths "Introduction to electrodynamics" est mentionné le théorème fondamental des gradients: https://en.wikipedia.org/wiki/Gradient_theorem : $\int_{\mathcal{C}} \vec{\nabla} V \cdot d\vec{l} = V(\vec{r}_B) - V(\vec{r}_A)$ où $\vec{r}_A$ et $\vec{r}_B$ désignent les vecteurs de départ et d'arrivée de la courbe $\mathcal{C}$, respectivement.

La démonstration de ce théorème est très facile en utilisant le théorème fondamental de l'analyse. Un corollaire de ce théorème est le suivant: $\oint_{\mathcal{C}} \vec{\nabla} V \cdot d\vec{l} = V(\vec{r}_A) - V(\vec{r}_A) = 0.$

Grâce à ce corollaire et à la loi de Lenz-Faraday (en l'absence de flux d'induction magnétique), on montre également facilement la loi des mailles de Kirchhoff. En effet, en partant de la loi de Lenz-Faraday et puisque le champ électrique $\vec{E}$ est, dans ce contexte, conservatif (i.e.: $\vec{E} = - \vec{\nabla} V$), il s'ensuit que
\begin{align}
&\oint_{\mathcal{C} = \mathcal{C}_1 \circ \ \mathcal{C}_2 \ \circ ...} \vec{E} \cdot d \vec{l} = 0 \ \textrm{ où les $\mathcal{C}_i$ correspondent aux différents branchages de mon circuit} \\
\iff & - \sum_{i} \int_{\mathcal{C}_i} \vec{\nabla} V \cdot d \vec{l} = 0 \\
\iff & \sum_{i} \Delta V_i = 0 .
\end{align}
Cela étant, j'aimerais étendre cette dernière formule de Kirchhoff (i.e.: $\sum_{i} \Delta V_i = 0$) en présence d'un flux d'induction magnétique variable. Intuitivement, si $L$ désigne l'inductance de ma bobine, je m'attends à ce que la formule de Faraday soit de la forme $\sum_{i} \Delta V_i = L \frac{dI(t)}{dt}$, où $I(t)$ est le courant qui parcourt le circuit. C'est ce que l'on rencontre dans la plupart des textbooks avec le terme d'induction magnétique dans le membre de gauche de l'équation (avec un signe négatif) et où les auteurs appellent cette formule faussement la loi de Kirchhoff.
Lorsque j'essaie de démontrer cette formule, je repars de l'équation de Lenz-Faraday
\begin{align}
&\oint_{\mathcal{C} = \mathcal{C}_1 \circ \ \mathcal{C}_2 \ \circ ...} \vec{E} \cdot d \vec{l} = - \frac{d}{dt} \iint_{\mathcal{S}} \vec{B} \cdot d\vec{S} = - L \frac{dI(t)}{dt} .
\end{align}
Là, je ne peux plus écrire $\vec{E} = - \vec{\nabla} V$ car, à cause du corollaire du thm fondamental des gradients, je retomberais dans le cas particulier d'une situation sans variation de flux d'induction magnétique. Comment poursuivre ma démonstration alors ? Car je suis persuadé que la formule $\sum_{i} \Delta V_i = L \frac{dI(t)}{dt}$ est valide puisqu'elle est utilisée dans tous les problèmes de montages RL, RLC et j'en passe.

Réponses

  • Il me sembe que le potentiel est défini par $\underline{E} = -\underline{\nabla} V$. Comment on le définirait sans cette équation ?
  • Tibsoo
    Modifié (November 2022)
    Justement, dans ce cas précis, le champ vectoriel $\vec{E}$ n'est plus conservatif. Il n'est donc plus question d'écrire $\vec{\nabla} \times \vec{E} = 0 \implies \exists V \ t.q. \vec{E} = - \vec{\nabla} V$ 

    D'ailleurs, le potentiel électrique n'est jamais défini selon l'équation $\vec{E} = - \vec{\nabla} V$ mais plutôt comme une intégrale $V(\vec{r}_A) = - \int_{\mathcal{O}}^{\vec{r}_A} \vec{E} \cdot d \vec{l}$. 
  • Bibix
    Modifié (November 2022)
    Oui, on a en fait $\underline{E} = - \underline{\nabla} V - \frac{\partial \underline{A}}{\partial t}$ avec $\underline{A}$ le potentiel-vecteur. Je m'en souviens maintenant.
  • Bibix a dit :
    Oui, on a en fait $\underline{E} = - \underline{\nabla} V - \frac{\partial \underline{A}}{\partial t}$ avec $\underline{A}$ le potentiel-vecteur. Je m'en souviens maintenant.
    C'est préférable de ne pas zapper $\bf{A}$, si on ne veut pas transgresser Maxwell $\nabla \times \bf{E} = -\dfrac{1}{c} \dfrac{\partial \bf{B}}{\partial t}$.
Connectez-vous ou Inscrivez-vous pour répondre.